Difference between revisions of "2005 AMC 10B Problems/Problem 5"

(Solution)
m
Line 1: Line 1:
{{empty}}
 
 
== Problem ==
 
== Problem ==
 
Brianna is using part of the money she earned on her weekend job to buy several equally priced CDs. She used one fifth of the money to buy one third of the CD's. What fraction of the money will she have left once she buys the CD's?
 
Brianna is using part of the money she earned on her weekend job to buy several equally priced CDs. She used one fifth of the money to buy one third of the CD's. What fraction of the money will she have left once she buys the CD's?
Line 12: Line 11:
 
*[[2005 AMC 10B Problems/Problem 6 | Next problem]]
 
*[[2005 AMC 10B Problems/Problem 6 | Next problem]]
 
*[[2005 AMC 10B Problems]]
 
*[[2005 AMC 10B Problems]]
 +
 +
[[Category: Introductory Algebra Problems]]

Revision as of 14:31, 4 January 2009

Problem

Brianna is using part of the money she earned on her weekend job to buy several equally priced CDs. She used one fifth of the money to buy one third of the CD's. What fraction of the money will she have left once she buys the CD's?

$\mathrm{(A)}\ {{{\frac{1} {5}}}} \qquad \mathrm{(B)}\ {{{\frac{1} {3}}}} \qquad \mathrm{(C)}\ {{{\frac{2} {5}}}} \qquad \mathrm{(D)}\ {{{\frac{2} {3}}} \qquad \mathrm{(E)}\ {{{\frac{4} {5}}}}$ (Error compiling LaTeX. Unknown error_msg)

Solution

Since she will use $\frac{1} {5}$ of her money to buy $\frac{1} {3}$ of the CD's, she will use $\frac{3} {5}$ of her money to buy all the CD's. Thus, she will have $1-\frac{3} {5}=\frac{2} {5}$ of her money left. Hence, the answer is $\boxed{C}$

See Also